hanliao1991
Thanks Received: 0
Forum Guests
 
Posts: 2
Joined: July 19th, 2013
 
 
 

Q22- This company will not be training

by hanliao1991 Fri Jul 19, 2013 5:22 pm

Could anyone please explain to me what answerA means and how does it make it possible that there will be shortage? Thanks a lot, I am having a difficult time understanding the answer!
User avatar
 
ManhattanPrepLSAT1
Thanks Received: 1909
Atticus Finch
Atticus Finch
 
Posts: 2851
Joined: October 07th, 2009
 
 
 

Re: Q22- This company will not be training

by ManhattanPrepLSAT1 Thu Jul 25, 2013 3:53 pm

Good question, thanks for bringing this one to the forum!

So, lets say the there are 10 airlines--each with a waiting list of 400 pilots. Each airline only needs 100 new pilots. It seems like the airline industry is fine, but wait! What if the same 400 pilots are on waiting lists at all 10 airlines.

The industry needs 1000 new pilots (100 per airline). But there are only 400 pilots (all on the same waiting lists at all 10 airlines). That's a big problem! The industry needs 1000 pilots, but there are only 400 pilots ready. Answer choice (A) points out this potential problem.

Incorrect Answers
(B) is out of scope, since this answer choice goes beyond the foreseeable future--the scope of the conclusion.
(C) is out of scope. Who cares about an age imbalance?
(D) strengthens the conclusion, rather than undermining it. We're looking to see if there could be a problem, not that everything has been accounted for.
(E) is irrelevant. The conclusion is about "this" airline and what others are doing does not matter.